Đến nội dung

Hình ảnh

BĐT AM-GM


  • Please log in to reply
Chủ đề này có 339 trả lời

#1
no matter what

no matter what

    Why not me

  • Thành viên
  • 397 Bài viết

*
Phổ biến

Mọi người cho mình bổ sung 1 cái note nho nhỏ nhé:Vì mình đã có 1 cái lịch khá bài bản cho topic này nên tạm thời mong mọi người tham gia XỬ LÝ các bài trong topic trước,khi nào phần kiến thức post hết,mọi người hãy thoải mái post bài ngoài.Mình thật sự xin lỗi vì sự bất tiện này và cũng mong mọi người cùng giúp để topic phát triển -Xin chân thành cảm ơn
Trước khi vào vấn đề chính ,mình xin có đôi lời như sau
1,**Topic về cơ bản là để định hướng các bạn đến với BĐT,vì vậy mong mọi người tôn trọng các bà toán trong topic,nếu ai thấy quá dễ cũng xin không đặt commen vào
2,**Trong topic,mọi người đều bình đẳng ,dù min hay mod vào topic đều phải cởi bở quân phục và vũ khí
3,**Hầu hết để giải BĐT,mọi người đều tìm cách trả lời cho câu hơi"tại sao" và" như thế nào",trong topic ,mình sẽ cùng mọi người tìm cách giải quyết,càng triệt để,càng chi tiết càng tốt
4,**Trong topic này ,mình không phải là chủ mà cũng như các bạn đều là người học,do kiến thức còn quá hạn hẹp(đặc biệt là về kĩ thuật cân bằng hệ số) nên mọi vấn đề chỉ có thể giải quyết nếu dc mọi người nhiệt tình giúp đỡ
5,**Mình cần lưu ý thêm là hầu hết các bài toán trên đây đều là của người khác ra đề,vì vậy,trên nguyên tắc không thể bó hep nó chỉ ở BĐT AM-GM,vì vậy ,sx có những BĐT cần tới cả C-S ... cũng nên,mong mọi người lươgj thứ về vấn đề này
6**,Nếu có nguồn bài,mình sẽ ghi rõ,tuy nhiên,mình chưa bao giờ có ý nghĩ rằng BĐT ở IMO sẽ khó hơn VMO ,sẽ khó hơn BĐTthi HSG tỉnh nào đó,...mong các bạn cũng nghĩ như mình để không tạo tâm lý làm bài
7,**Các bài toán trong topic có thể các bạn sẽ giải dễ dàng nhưng nếu gặp 1 bài BĐT bất kì,nhiêud khi bạn lại sẽ rát lúng túng do không phải pải sử dụng cái gì à như thế nào.Các bạn khoan chớ lo về vấn đề ấy,để khắc phục ,các bạn cần nắm vững những điều sau đã
8,**Các bạn đừng ngần ngại phản hồi vào topic BẤT CỨ ĐIỀU GÌ CÁC BẠN BĂN KHOĂN ,mọi người sẽ cùng thảo luận ,đừng để cái không biết của bây giờ đè nát cái thông mnih của ngày sau
9,**Mọi sai sót trong quá trình mong mọi người lượng thứ :icon6:
Trên tinh thần trên ,trước hết mình xin chỉ nêu ra 2 BĐT mà mình cho là cơ bản nhất AM-GM và CAUCHY-SCHWARZ )
Vè lý thuyết và phát biểu ,có lẽ mọi người đều đã nắm rõ,và vì đã có ở topic http://diendantoanho...c-cực-trị-thcs/ nên mình xin không nêu lại
trên đây,mình xin nêu ra một số hệ quả trực tiếp vào bài toán định hướng của 2 bđt trên
*Hệ quả BĐT AM-GM(xét với các số không âm nhé)
1,$(a+b+c)^2\geq 3(ab+bc+ca)$
2,$a^3+b^3+c^3\geq 3abc$
3,$(ab+bc+ca)^2\geq 3abc(a+b+c)$
4,$\frac{1}{a}+\frac{1}{b}\geq \frac{4(=2^2)}{a+b}$ hay $\frac{1}{a+b}\leq \frac{1}{4}(\frac{1}{a}+\frac{1}{b})$ (tương tự với n số,bạn cũng có thể tổng quát lên dễ dàng )
*BĐT phụ :Không chỉ với AM-GM,BĐT phụ còn có tầm ảnh hương tới rất nhiều bài toán BĐT khác(đến nỗi mà đã có 1 topic riêng rất hay về phần này http://diendantoanho...-dẳng-thức-phụ/
mọi người tham khảo link trên,học thuộc dc thì càng tốt :lol:
*Hệ quả BĐT C-S(CAUCHY-SCHWARZ )
1, 2 hệ quả rất quan trọng của BĐT C-S là (Schwars và Mincowski đều dc trình bày ở link mình đã đưa đàu bài )
*Bài toán định hướng
1,$\sqrt{\frac{a}{b+c}}\geq \frac{2a}{a+b+c}$
-Chứng minh
BĐT tương đương $\sqrt{a}(a+b+c)\geq 2a\sqrt{b+c}$ hay $a+(b+c)\geq 2\sqrt{a.(b+c)}$ (luuôn đúng theo AM-GM 2 số)
lời giải trên liệu có gì không tự nhiên không mọi người nhỉ
(cái ta để tam ở đây là BĐT trên là 1 cách ta khử căn thức để làm BĐT đỡ "vất vả" hơn )
2,$\frac{a+b+c+d}{\sqrt[4]{abcd}}+\frac{16}{(a+b)(b+c)(c+d)(d+a)}\geq 5$
-Chứng minh :chú ý rằng $\frac{a+b+c+d}{\sqrt[4]{abcd}}=\frac{a+b}{2\sqrt[4]{abcd}}+\frac{b+c}{\sqrt[4]{abcd}}+\frac{c+d}{\sqrt[4]{abcd}}+\frac{d+a}{\sqrt[4]{abcd}}$
đến đây, nếu trở ại bài toán,ta chỉ cần áp dụng BĐT AM-GM cho 5 số là xong
(cái mình muốn nói ở đây là viêc tách hạng tử
nhiều bạn chắc chắn sẽ thắc mắc tại sao lại tách như trên .Trong tầm kiến thức ở AM-GM(tức là ta chỉ xem xét các khả năng giải bằng AM-GM cho bài toán trên ),mình xin giải thích như sau
+,nếu áp dụng trực tiếp AM-GM ,chắc chắn sẽ không đem lại kết quả rồi
+,chú ý rằng bài toán trên không hề cho đk ẩn(ngoài không âm)nên ta khả dĩ không thể áp dụng AM-GM ĐỂ RỒI ĐƯA VẾ TRÁI VỀ 1 BIỂU THỨC CÒN ẨN
+,từ đánh giá thứ 2,ta buộc phải tìm cách triệt tiêu hoàn toàn tất cả ẩn sau khi đã áp dụng AM-GM
tức là phải tách 1 trong 2 hạnh tử trên ra thành tổng của 1 số hạng tử náo đó
+,Phân thức $\frac{16abcd}{(a+b)(b+c)(c+d)(d+a)}$ tuyệt vời này chắc cũng chẳng ai tìm cách tách nó,vì vậy ta tìm cách tách phân thức $\frac{a+b+c+d}{\sqrt[4]{abcd}}$
+'dựa vào phân thức cố định trên,ta phải tách phân thức $\frac{a+b+c+d}{\sqrt[4]{abcd}}$ thành tổng các phân thức có mẫu là $k(a+b),k(b+c)..$ để triệt tiêu mẫu(số pphân thức tách được là 4 )
+,chọn k= 1(vì sao?chẳng vì sao cả,nếu không dc ta lại chọn số khác ,cho dễ tihs thôi) ta dc 1 dạng $\frac{a+b}{x}+\frac{b+c}{y}+\frac{c+d}{z}+\frac{d+a}{t}$
+,chọn x=y=z=t ,ta dc tổng trên bằng $\frac{2(a+b+c+d)}{x}$ vậy hiển nhiên $x=2\sqrt[4]{abcd}$
đến đây xem như ta đã hoàn thành trọn vẹn bài toán
3,$(a+b+c)(ab+bc+ca)\leq \frac{8}{9}(a+b)(b+c)(c+a)$
(mình xin nêu kái quát về phép đổi biến p.q.r,nếu có dk mình sẽ giới thiệu ở BĐT schur, hoặc kĩ thuật đặt ẩn phụ )
để giảm bớt khối lượng tính toán,trong trường hợp đủ bộ,ta có thể đặt
$a+b+c=p;ab+bc+ca=q;abc=r$,khi đó hiển nhiên là $p\geq 3r;q\geq 3p;p^2\geq 3q,q^2\geq 3pr$
(còn rất nhiều BĐT nữa mình sẽ giới thêu sau và cũng khuyến khích mọi người thử xây dựng)
*1 hằng đẳng thức quý vô bờ bến mà mọi người cần nhớ là $(a+b)(b+c)(c+a)= (a+b+c)(ab+bc+ca)-abc$
chứng minh thuần bằng đại số
trở lại bài toán ,BĐT viết lại như sau $9r\leq pq$ và điều này là hiển nhiên
4,$\frac{a^2}{b+c}+\frac{b^2}{c+a}+\frac{c^2}{a+b}\geq \frac{a+b+c}{2}$
Mặc dù BĐT có thể giải hết sức đơn giản bằng C-S tuy nhiên cái mình muốn nói ở đây(và cũng là cái mình yếu nhất)là khử mẫu-điểm rơi
+,trước tiên ta để ý là không thể cộng trực tiếp các số hạng ở VT lại,làm như vậy quá "mạo hiểm" :excl:
+,tư tưởng của ta là khử mẫu cho dễ tính toán và AM-GM có thể làm điều này giúp bạn
$\frac{a^2}{b+c}+\frac{b+c}{x}\geq 2\sqrt{\frac{a^2.(b+c)}{x.(b+c)}}= 2\frac{a}{\sqrt{x}}$
cái quan trọng giờ là con x ấy là con lào ?ta không dc chọn mò nó,vì sao?vì theo AM-GM thì các số hạng được vinh dự tham gia áp dụng phải bằng nhau thì mới có đẳng thức ,đây cũng là mấu chốt để tìm x,
+,ta phải có $\frac{a^2}{b+c}=\frac{b+c}{x}$,(1)chú ý hơn 1 tí nữa,trong hầu hết các BĐT sử dụng AM-GM hay C-S(ta xét 3 biến a,b,c) đẳng thức xảy ra khi a=b=c hoặc (a=b,c=0)và các hoán vị,trong bài toán này nếu thay a=b=c,ta sẽ có đẳng thức(2)
từ (1) và(2) ta đễ tìm dc x,phần còn lại bạn trình bày nốt nhé

Bài viết đã được chỉnh sửa nội dung bởi no matter what: 24-11-2012 - 23:33


#2
no matter what

no matter what

    Why not me

  • Thành viên
  • 397 Bài viết

*
Phổ biến

Bây giờ mình xin trình bày 1 số ĩ thuật nhỏ về BĐT AM-GM
1,Kĩ thuật đánh giá trung bình cộng-trung bình nhân
Về phần này,mình sẽ không nêu kiế thức,các bạn có thể rút ra phương pháp từ các VD sau
VD1,chứng minh với $a>c>0,b>c>0$ ta có BĐT sau
$\sqrt{c(a-c)}+\sqrt{c(b-c)}\leq \sqrt{ab}$
Giải :Đứng trước 1 bài toán như vậy,bạn sẽ làm gì?Bình phương lên -đó cũng là 1 cách .Tuy nhiên,hãy xem xét cách sau
BĐT tương đương $\sqrt{\frac{c(a-c)}{ab}}+\sqrt{\frac{c(b-c)}{ab}}\leq 1$
Áp dụng AM-GM ta có :VT
$\leq \frac{1}{2}(\frac{c}{b}+\frac{a-c}{a})+\frac{1}{2}(\frac{c}{a}+\frac{b-c}{b})$=
=$\frac{1}{2}(\frac{a}{a}+\frac{b}{b})=1$ (dpcm)
so với phương pháp bình phương,đây là 1 lời giải gọn hơn phải không các bạn :icon6:
VD2:Chứng minh với moi số thực dương a,b,c,x,y,z ta có
$\sqrt[3]{(a+y)(b+y)(c+z)}\geq \sqrt[3]{abc}+\sqrt[3]{xyz}$
Giải:bạn sẽ làm gì với BĐT trên ?Lập phương ư?đó cũng là 1 cách,nhưng bạn phải thấy là lập phương 1 tổng không dễ dàng như khi bình phương 1 tổng.Hãy chú ý tới cách làm sau
BĐT tương đương $\sqrt[3]{\frac{abc}{(a+x)(b+y)(c+z)}}+\sqrt[3]{\frac{xyz}{(a+x)(b+y)(c+z)}}\leq 1$
Theo AM-GM 3 số,ta có VT$\leq \frac{1}{3}(\frac{a}{a+x}+\frac{b}{b+y}+\frac{c}{c+z})+\frac{1}{3}(\frac{x}{a+x}+\frac{y}{b+y}+\frac{z}{c+z})=1$
1 lời giải rất gọn phải không các bạn?
Sau đây là 1 số bài tập nho nhỏ cho các bạn :lol:
1,Chứng minh với mọi a,b,c không âm
$\sqrt{ab}+\sqrt{cd}\leq \sqrt{(a+c)(b+d)}$
2,Chứng mnih với mọi a,b,c không âm
$\sqrt[3]{abc}+1\leq \sqrt[3]{(a+1)(b+1)(c+1)}$
3,(tổng quát bài 1 lên) với 2n số không âm
$\sqrt[n]{a_{1}a_{2}....a_{n}} +\sqrt[n]{b_{1}b_{2}...b_{n}}\leq \sqrt[n]{(a_{1}+b_{1})(a_{2}+b_{2})...(a_{n}+b_{n})}$
4,(Crech and Slovak 2000) :Chứng minh BĐT sau với mọi a,b ko âm
$\sqrt[3]{\frac{a}{b}}+\sqrt[3]{\frac{b}{a}}\leq \sqrt[3]{2(a+b)(\frac{1}{a}+\frac{1}{b})}$

Bài viết đã được chỉnh sửa nội dung bởi no matter what: 23-11-2012 - 13:37


#3
no matter what

no matter what

    Why not me

  • Thành viên
  • 397 Bài viết
Nói thêm 1 tí,tuy rằng đây không phải là kĩ thuật có ứng dụng rộng,tuy nhiên cũng xin đưa ra để lỡ có dính thì bạn cũng đừng lúng túng
Kĩ thuật đánh giá mẫu
Ta xét VD sau :Cho a,b,c là độ dài 3 cạnh của tam giác,chứng minh BĐT sau
$\sqrt{\frac{a}{b+c-a}}+\sqrt{\frac{b}{c+a-b}}+\sqrt{\frac{c}{a+b-c}}\geq 3$
Hiển nhiên BĐT trên có thể chứng minh trực tiếp bằng AM-GM 3 số rất gọn,tuy nhiên vì là bài giứoi thiệu phương pháp nên mọi người chú ý tới đánh giá sau
Theo AM-GM ,ta có $2\sqrt{\frac{b+c-a}{a}}\leq \frac{b+c-a}{a}+1=\frac{b+c}{a}\Rightarrow \sqrt{\frac{a}{b+c-a}}\geq \frac{2a}{b+c}$
đánh giá tương tự ta đã có lời giải cho bài toán
Vì dạng này không khó nên dành lại VD sau cho các bạn
1,(Romania 1993) Tìm hằng số a lớn nhất để BĐT sau đúng với mọi x,y,z dương
$\frac{x}{\sqrt{y^2+z^2}}+\frac{y}{\sqrt{x^2+z^2}}+\frac{z}{\sqrt{x^2+y^2}}\geq a$
Đến phần chính :Kĩ thuật đặt ẩn phụ
Về kĩ thuật này ,trong sách của anh Cẩn cũng đã nói khá rõ,vì vậy mình sẽ lấy các VD ở ngoài và ở phài luyện thêm chưa được giải
VD1,Chứng minh với mọi a,b,c dương ,ta có BĐT sau
$\frac{ab}{c}+\frac{bc}{a}+\frac{ca}{b}\geq \sqrt{3(a^2+b^2+c^2)}$
Bìa toán chưa đòi hởi ta biến đổi để tìm ẩn phụ khá đơn giản
Đặt $\frac{ab}{c}=x;\frac{bc}{a}=y;\frac{ca}{b}=z$,khi đó ta có ngay $xy=b^2,yz=c^2;zx=a^2$
Vậy ta cần CM $x+y+z\geq \sqrt{3(xy+yz+zx)}$
Tuy nhiên BĐT cuối là hệ quả trực tiếp của AM-GM nên ta có dpcm
VD2,Chứng mnih với mọi a,b,c dương ta cs
$\sqrt{\frac{(a^2+bc)(b+c)}{a(b^2+c^2)}}+\sqrt{\frac{(b^2+ca)(c+a)}{b(c^2+a^2)}}+\sqrt{\frac{(c^2+ab)(a+b)}{c(a^2+b^2)}}\geq 3\sqrt{2}$
Giải: Xem phát biểu của bài toán trên,nhiều người chắc cũng hơi ngán :wacko: ,tuy nhiên bạn hãy chú ý là đó chỉ là 1 cách lừa tình mà thôi
Chú ý rằng $(a^2+bc)(b+c)=b(c^2+a^2)+c(a^2+b^2)$,BĐT tuyệt vời của chúng ta viết lại được thành
$\sqrt{\frac{b(c^2+a^2)+c(a^2+b^2)}{a(b^2+c^2)}}+\sqrt{\frac{c(a^2+b^2)+a(b^2+c^2)}{b(c^2+a^2)}}+\sqrt{\frac{a(b^2+c^2)+b(c^2+a^2)}{c(a^2+b^2)}}\geq 3\sqrt{2}$
Đến đây ta sẽ đặt $a(b^2+c^2)=x;b(c^2+a^2)=y;c(a^2+b^2)=z$,viết BĐT thành
$\sqrt{\frac{x+y}{z}}+\sqrt{\frac{y+z}{x}}+\sqrt{\frac{z+x}{y}}\geq 3\sqrt{2}$
BĐT cuối có thể chứng minh dễ dàng bằng AM-GM 3 số,xin dành cho các bạn
*Bây giờ mình xin tập trung ní vào cái mình chú ý ở kĩ thuật đặt ẩn phụ
Kĩ thuật đặt ẩn phụ đặc trưng cho 1 lớp BĐT mà sẽ rất khó giải hoặc thậ chí không giải được nếu không có ẩn phụ
Ngoài điều kiện các ẩn đều dương,1 số BĐT còn có thêm các điều kiện sau
+,Điều kiện $abc=1$
Ta chú ý rằng ,dk đã cho cũng đồng nghĩa rằng tồn tại x,y,z dương sao cho $a=\frac{x}{y};b=\frac{y}{z};c=\frac{z}{x}$
Ta sẽ ứng dụng nhận xét này giải VD sau
(IMO 2000),Chứng mnih với mọi a,b,c dương ta có
$(a-1+\frac{1}{b})(b-1+\frac{1}{c})(c-1+\frac{1}{a})\leq 1$
Giải: Cách giải là dựa vào đánh giá trên,bạn tự mình giải chắc sẽ thú vị hơn nhỉ <_<
+ Điều kiện $xyz=x+y+z+2$
Điều kiện đó cũng đồng nghĩa với việc tồn tại a,b,c sao cho $x=\frac{b+c}{a};y=\frac{c+a}{b};z=\frac{a+b}{c}$
Phải chăng bạn nghĩ là mình đã áp đặt các ẩn phụ trên vào và nói là tồn tại?không đâu,mình xin giải thích trường hợp này
$xyz=x+y+z+2\Leftrightarrow$
$\Leftrightarrow xyz+(xy+yz+zx)+(x+y+z)+1=2(x+y+z)+xy+yz+xz +3$
$\Leftrightarrow (x+1)(y+1)(z+1)=(x+1)(y+1)+(y+1)(z+1)+(z+1)(x+1)$
$\Leftrightarrow \frac{1}{1+x}+\frac{1}{1+y}+\frac{1}{1+z}=1$
Vậy,nếu đặt $a=\frac{1}{1+x},b=\frac{1}{1+y},c=\frac{1}{1+z}$
khi đó $a+b+c=1$ và $x=\frac{1-a}{a}=\frac{b+c}{a}$,tương tự với y,z
Và đy là 1 VD mẫu :
Với mọi x,y,z dương thỏa mãn $xyz=x+y+z+2$ chứng minh
$xy+yz+zx\geq 2(x+y+z)$
Giải: điều kiện đề bài cho phép ta biến $(x;y;z) \to \(\frac{b+c}{a};\frac{c+a}{b};\frac{a+b}{c})$
ta cần chứng minh BĐT $\sum \frac{(b+c)(c+a)}{ab}\geq 2\sum \frac{b+c}{a}$
khai triển trực tiếp BĐT tương đương
$a^3+b^3+c^3+3abc\geq a^2b+ab^2+b^2c+bc^2+c^2a+ca^2$
Đây chính là BĐT schur (các bạn có thể tìm lời giải trên google-xin lỗi vì sự quá đà này)
+,Đièu kiện $xy+yz+zx+2xyz=1$
Điều kiện này cho ta biến $(x,y,z) \to (\frac{a}{b+c};\frac{b}{c+a};\frac{c}{a+b})$ mọi người nhé (~~)
+,Điều kiện $ab+bc+ca+abc=4$
Điều kiện này cho ta biến $(a,b,c) \to (\frac{2x}{y+z},\frac{2y}{z+x},\frac{2z}{x+y})$
*Mình xin dừng phần này ở đây đã
Một số bài tập nho nhỏ cho các bạn
1, chứng minh với mọi a,b,c không âm có $ab+bc+ca+abc=4$ thì
$a+b+c\geq ab+bc+ca$(VMO 1996)
2,Chứng minh với mọi a,b,c dương có $ab+bc+ca+2abc=1$ thì
$2(a+b+c)+1\geq 32abc$
3,chứng minh với mọi số thực dương a,b,c có tích bằng 1 thì
$\frac{a+b}{c}+\frac{b+c}{a}+\frac{c+a}{b}+6\geq 2(a+b+c+\frac{1}{a}+\frac{1}{b}+\frac{1}{c})$
4,Chứng minh với mọi a,b,c không âm có tích bằng 1
$\frac{a}{b^2(c+1)}+\frac{b}{c^2(a+1)}+\frac{c}{a^2(b+1)}\geq \frac{3}{2}$

Bài viết đã được chỉnh sửa nội dung bởi no matter what: 23-11-2012 - 13:40


#4
Oral1020

Oral1020

    Thịnh To Tướng

  • Thành viên
  • 1225 Bài viết
Sai thôi nhé mấy anh
Cho em làm bài 1:Không dùng $AM-GM$
Ta có $\sqrt{(a+c)(b+d)} \ge \sqrt{ab}+\sqrt{cd}$
$\Leftarrow (a+c)(b+d) \ge (\sqrt{ab}+\sqrt{cd})^2$(luôn đúng đối với $Cauchy-Schwarz$)

"If I feel unhappy,I do mathematics to become happy.


If I feel happy,I do mathematics to keep happy."

Alfréd Rényi

Hình đã gửi


#5
no matter what

no matter what

    Why not me

  • Thành viên
  • 397 Bài viết

Sai thôi nhé mấy anh
Cho em làm bài 1:Không dùng $AM-GM$
Ta có $\sqrt{(a+c)(b+d)} \ge \sqrt{ab}+\sqrt{cd}$
$\Leftarrow (a+c)(b+d) \ge (\sqrt{ab}+\sqrt{cd})^2$(luôn đúng đối với $Cauchy-Schwarz$)

cách chứng minh của em thì hoàn toàn không sai nhưng mục đích chính của anh là rèn luyện sử dụng cho thành thạo AM-GM ,đồng thời trong bài này,ta dùng AM-GM để chứng inh C-S :icon6:

#6
Zony Nguyen

Zony Nguyen

    Đốt Lửa

  • Thành viên
  • 123 Bài viết
Anh Quang à ! Anh có lòng vậy thì anh chém hộ em luôn cái bài này nha . Lớp 8 thôi ? :lol:
Câu 1 :
Cho $a,b,c > 0 ; a+b+c \leq 1$ . Chứng minh rằng :
$\frac{1}{a^{2}+2bc} + \frac{1}{b^{2}+2ca} + \frac{1}{c^{2}+ 2ab}$ $\frac{1}{a^{2}+2bc} + \frac{1}{b^{2}+2ca} + \frac{1}{c^{2}+ 2ab}\geq 9$
Câu 2 :
Cho $a;b;c \geq 0 ; a+b+c =1$
Chứng minh rằng :
a, $\sqrt{a+b} + \sqrt{b+c} +\sqrt{c+a} \leq \sqrt{6}$
b, $\sqrt{a+1} + \sqrt{b+1} + \sqrt{c+1}< 3,5$
Còn nhiều bài nữa cơ thực ra thì em cũng có biết chút về cái này tôi Cho em làm một chân nha !
Em chưa học cái này nhưng mong Anh chị chỉ giúp em .
À ! mong mấy anh chị chỉ cho em xem có mấy cách chứng minh bất đẳng thức . Lấy ví dụ cho em một thể.Mấy bài trên thì cũng dễ em vừa làm xong mong anh chị làm cho em xem em làm có đúng không nha . Em cảm ơn > :icon6: :icon12: :icon12: :icon12: :icon12:

Bài viết đã được chỉnh sửa nội dung bởi DUY MAM: 21-11-2012 - 19:36

Chúc anh em luôn vui vẻ ! nhiều sức khỏe ! Nhận nhiều like

#7
Oral1020

Oral1020

    Thịnh To Tướng

  • Thành viên
  • 1225 Bài viết
Câu 1 hình như đề sai nhé phải $\ge 18$ mới đúng(Dự đoán khi $a=b=c\dfrac{1}{3}$
Nếu thế thì giải nhé:
Áp dụng $C-S$,ta có $VT \ge \dfrac{36}{2(a+b+c)^2}$
Ta có $2(a+b+c)^2 \le 2$ nên $\dfrac{36}{2(a+b+c)^2} \ge 18$dpcm
Bài 2:
Áp dụng $AM-GM$ cho $(a+b)$ và $\dfrac{2}{3}$,ta có
$\dfrac{a+b+\dfrac{2}{3}}{2} \ge \sqrt{\dfrac{2}{3}(a+b)}$
Tương tự,ta có $a+b+c+1 \ge \sum \sqrt{\dfrac{2}{3}(a+b)}$
$\Leftrightarrow \dfrac{2}{\sqrt{\dfrac{2}{3}}} \ge \sum \sqrt{a+b}$
do $\dfrac{2}{\sqrt{\dfrac{2}{3}}}=\sqrt{6}$nên ta có dpcm
Câu b)thì bạn Áp dụng $AM-GM$ cho $a+1$ và $\dfrac{4}{3}$

Bài viết đã được chỉnh sửa nội dung bởi Oral1020: 21-11-2012 - 19:53

"If I feel unhappy,I do mathematics to become happy.


If I feel happy,I do mathematics to keep happy."

Alfréd Rényi

Hình đã gửi


#8
no matter what

no matter what

    Why not me

  • Thành viên
  • 397 Bài viết

Anh Quang à ! Anh có lòng vậy thì anh chém hộ em luôn cái bài này nha . Lớp 8 thôi ? :lol:
Câu 1 :
Cho $a,b,c > 0 ; a+b+c \leq 1$ . Chứng minh rằng :
$\frac{1}{a^{2}+2bc} + \frac{1}{b^{2}+2ca} + \frac{1}{c^{2}+ 2ab}$ $\frac{1}{a^{2}+2bc} + \frac{1}{b^{2}+2ca} + \frac{1}{c^{2}+ 2ab}\geq 9$
Câu 2 :
Cho $a;b;c \geq 0 ; a+b+c =1$
Chứng minh rằng :
a, $\sqrt{a+b} + \sqrt{b+c} +\sqrt{c+a} \leq \sqrt{6}$
b, $\sqrt{a+1} + \sqrt{b+1} + \sqrt{c+1}< 3,5$
Còn nhiều bài nữa cơ thực ra thì em cũng có biết chút về cái này tôi Cho em làm một chân nha !
Em chưa học cái này nhưng mong Anh chị chỉ giúp em .
À ! mong mấy anh chị chỉ cho em xem có mấy cách chứng minh bất đẳng thức . Lấy ví dụ cho em một thể.Mấy bài trên thì cũng dễ em vừa làm xong mong anh chị làm cho em xem em làm có đúng không nha . Em cảm ơn > :icon6: :icon12: :icon12: :icon12: :icon12:

em xem lại đề bài 1 xem hình như thừa thì phải
đề đúng là $\frac{1}{a^2+2bc}+\frac{1}{b^2+2ca}+\frac{1}{c^2+2ab}\geq 9$ phải không em?

#9
no matter what

no matter what

    Why not me

  • Thành viên
  • 397 Bài viết

Câu 1 đề sai nhé phải $\ge 18$ mới đúng(Dự đoán khi $a=b=c\dfrac{1}{3}$
Nếu thế thì giải nhé:
Áp dụng $C-S$,ta có $VT \ge \dfrac{36}{2(a+b+c)^2}$
Ta có $2(a+b+c)^2 \le 2$ nên $\dfrac{36}{2(a+b+c)^2} \ge 18$9dpcm

em đừng quá áp dụng C-S nhé,đây là box luyện AM-GM mà hj,nếu có đk anh sẽ mở box C-S sau

Anh Quang à ! Anh có lòng vậy thì anh chém hộ em luôn cái bài này nha . Lớp 8 thôi ? :lol:

sao em biết anh nhỉ :icon6:

#10
Zony Nguyen

Zony Nguyen

    Đốt Lửa

  • Thành viên
  • 123 Bài viết
Thôi cho em làm nốt rồi ai muốn xóa thì xóa vậy ! :( :angry: :excl: :excl: :lol:
Câu 1 :
Bài làm :
Đặt $a^{2} + 2bc = x;c^{2} + 2ab = z:b^{2} + 2ca = y$
Khi đó $x+y+z = (a+b+c)^{2} \leq 1$
Vậy bài toán sẽ chuyển sang dạng :
$x,y,z > 0 ; x+y+z \leq 1$
Chứng minh rằng : $\frac{1}{x} + \frac{1}{y}+ \frac{1}{z} \geq 9$
Áp dụng bất đẳng thức Côsi là xong thôi.
Đề đúng rồi Mọi người xem em làm đúng chưa vậy . Duy nè anh.
Chúc anh em luôn vui vẻ ! nhiều sức khỏe ! Nhận nhiều like

#11
minhtuyb

minhtuyb

    Giả ngu chuyên nghiệp

  • Thành viên
  • 470 Bài viết

3,(tổng quát bài 1 lên) với 2n số không âm
$\sqrt[n]{a_{1}a_{2}....a_{n}} +\sqrt[n]{b_{1}b_{2}...b_{n}}\leq \sqrt[n]{(a_{1}+b_{1})(a_{2}+b_{2})...(a_{n}+b_{n})}\(*)$

Nhìn phát thấy ngay là $Holder$ Hình đã gửi
---
-Nếu $\exists a_i+b_i=0\Rightarrow a_i=b_i=0\Rightarrow VT(*)=VP(*)$
-Nếu các biến đều dương: áp dụng BĐT $AM-GM$ cho $n$ số, ta có:
$$\dfrac{a_1}{a_1+b_1}+\dfrac{a_2}{a_2+b_2}+...+\dfrac{a_n}{a_n+b_n}\ge \dfrac{n\sqrt[n]{a_1a_2...a_n}}{(a_{1}+b_{1})(a_{2}+b_{2})...(a_{n}+b_{n})}\\
\dfrac{b_1}{a_1+b_1}+\dfrac{b_2}{a_2+b_2}+...+\dfrac{b_n}{a_n+b_n}\ge \dfrac{n\sqrt[n]{b_1b_2...b_n}}{(a_{1}+b_{1})(a_{2}+b_{2})...(a_{n}+b_{n})}$$
Cộng vế với vế của 2 BĐT cùng chiều trên, sau vài bước biến đổi ta có ĐPCM. Dấu bằng xảy ra chẳng hạn khi $a_1=a_2=...=a_n;b_1=b_2=...=b_n\ \square$
---
Hai bài trên là hệ quả trực tiếp của BĐT này
Phấn đấu vì tương lai con em chúng ta!

#12
no matter what

no matter what

    Why not me

  • Thành viên
  • 397 Bài viết
mọi người ta tập trung lại 1 tí nhé,mình post bài có kèm theo bài tập là muốn mọi người vận dụng thẳng các kiến thức trên vào bài tập cho vững luôn,dĩ nhiên mình không nề nà gì việc đăng thêm bài mới,nhưng mình nghĩ,việc đó nên để khi ta đã giải quyết trọn vẹn các bài tập
thân
tình hình là ta còn nhiều bài tồn đọng quá
ĐỀ NGHỊ
bài 4 phần 1(kiến thức đã nêu,mong mọi người triệt để áp dụng :wacko:
với mọi a.b không âm,chứng minh
$\sqrt[3]{\frac{a}{b}}+\sqrt[3]{\frac{b}{a}}\leq \sqrt[3]{2(a+b)(\frac{1}{a}+\frac{1}{b})}$

#13
no matter what

no matter what

    Why not me

  • Thành viên
  • 397 Bài viết
+,Kết hợp BĐT AM-GM và BĐT phụ-Kĩ thuật đánh giá mẫu
Như đã nói,BĐT phụ là 1 phần cao cả của BĐT,ta sẽ xem nó có ứng dụng rộng rãi như thế nào trong BĐT AM-GM nhé
VD1:(USA MO 1998):Chứng minh với mọi a,b,c thực dương
$\frac{1}{a^3+b^3+abc}+\frac{1}{b^3+c^3+abc}+\frac{1}{c^3+a^3+abc}\leq \frac{1}{abc}$
Giải: BĐT phụ cần áp dụng là $a^3+b^3\geq ab(a+b)$
Áp dụng BĐT trên vào bài toán,ta dễ có đáp án (xin nhường cho các bạn :icon6: )
*Chú ý:tư tưởng của cách làm trên là đưa về đồng bậc
và đây là 1 VD tương tự ---
(IMO Shortlist 1996) Chứng minh với mọi x,y,z dương có tích bằng 1
$\frac{xy}{x^5+y^5+xy}+\frac{yz}{y^5+z^5+yz}+\frac{zx}{z^5+x^5+zx}\leq 1$
VD2 :(APMO 1998) Chứng minh với mọi a,b,c dương
$(1+\frac{x}{y})(1+\frac{y}{z})(1+\frac{z}{x})\geq 2+\frac{2(x+y+z)}{\sqrt[3]{xyz}}$(*)
Ta có thể coi BĐT $\frac{x}{y}+\frac{y}{z}+\frac{z}{x}\geq \frac{x+y+z}{\sqrt[3]{xyz}}$ (**)là BĐT phụ của BĐT trên không nhỉ ?
Dĩ nhiên BĐT(**) được suy ra trực tiếp từ BĐT(*)
Chứng minh BĐT (*) được suy ra trực tiếp từ đánh giá sau
$\frac{x}{y}+\frac{x}{y}+\frac{y}{z}\geq 3\sqrt[3]{\frac{x^2}{yz}}= \frac{3x}{\sqrt[3]{xyz}}$
VD3:(OLIMPIC 30-4)Chứng minh với mọi a,b,c dương có tổng bình phương bằng 1,ta có BĐT
$\frac{a}{b^2+c^2}+\frac{b}{c^2+a^2}+\frac{c}{a^2+b^2}\geq \frac{3\sqrt{3}}{2}$(1)
Và BĐT phụ của BĐT trên là
với mọi a dương ,BĐT sau đúng$a(1-a^2)\leq \frac{2}{3\sqrt{3}}$ (2)
Ta sẽ chứng minh BĐT (2)
$BDT\Leftrightarrow a^2(1-a^2)^2\leq \frac{4}{27}$
Áp dụng AM-GM ,ta có
$a^2(1-a^2)^2=\frac{1}{2}.2a^2.(1-a^2)(1-a^2)\leq \frac{1}{2}\left [ \frac{2a^2+(1-a^2)+(2-a^2)}{3} \right ]^3=\frac{4}{27}$
BĐT (1) đã được chứng minh
Sau đây là lời giải trọn vẹn
Vì $a^2+b^2+c^2=1$ nên ta viết BĐT thành$\frac{a}{1-a^2}+\frac{b}{1-b^2}+\frac{c}{1-c^2}\geq \frac{3\sqrt{3}}{2}(a^2+b^2+c^2)$
Do đó,ta chỉ cần chứng minh $\frac{a}{1-a^2}\geq \frac{3\sqrt{3}}{2}a^2$ hay $\frac{1}{a(1-a^2)}\geq \frac{3\sqrt{3}}{2}$
Nhưng BĐT đó đã được giải quyết triệt để :nav:
Hơn nữa,ta còn có BĐT sau (xin dành cho các bạn)
Chứng mnih với mọi a,b,c,d ,e, dương có $a^4+b^4+c^4+d^4+e^4=1$
$\frac{a^3}{b^4+c^4+d^4+e^4}+\frac{b^3}{a^4+c^4+d^4+e^4}+\frac{c^3}{d^4+e^4+a^4+b^4}+\frac{d^3}{e^4+a^4+b^4+c^4}+\frac{e^3}{a^4+b^4+c^4+d^4}\geq \frac{5\sqrt[4]{5}}{4}$
Đây là 1 phần rộng và đòi hỏi lượng kiến thức nhất định,sau đây mình xin đưa ra 1 list các BĐT phụ thường được dùng
1,$(a+b)(b+c)(c+a)\geq \frac{8}{9}(a+b+c)(ab+bc+ca)$
2,$ab+bc+ca\geq \sqrt{3abc(a+b+c)}$
3,$\frac{1}{(1+x)^2}+\frac{1}{(1+y)^2}\geq \frac{1}{1+xy}$
4,$\frac{ab}{c}+\frac{bc}{a}+\frac{ca}{b}\geq a+b+c$
5,$\frac{\sqrt{2a-1}}{a}\leq 1$
6,$abc\geq (a+b-c)(b+c-a)(c+a-b)$
7,$\frac{a^2}{b^2}+\frac{b^2}{c^2}+\frac{c^2}{a^2}\geq \frac{a}{b}+\frac{b}{c}+\frac{c}{a}$
8,$a^3+b^3\geq ab(a+b)$
9,(C-S 2 số)$\sqrt{(a+c)(b+d)}\geq \sqrt{ac}+\sqrt{bd}$
10,Schur: $a^3+b^3+c^3+3abc\geq a^2(b+c)+b^2(c+a)+c^2(a+b)$
*Kĩ thuật đánh giá mẫu
+,Khử căn:Các BĐT chứa căn thứa ở mẫu(có thể cả tử) luôn làm ta khổ sở vì khó xác định được cahs giải,và tư tưởng của ta là làm mất căn thức,sau đây,ta sẽ đến với 1 số VD sau
a,Nhân 1 lượng vừa đủ vào mẫu
Xét VD : Vói mọi a,b,c không âm thỏa mãn $a^2+b^2+c^2=2(ab+bc+ca)$,ta có
$\sqrt{\frac{ab}{a^2+b^2}}+\sqrt{\frac{bc}{c^2+c^2}}+\sqrt{\frac{ca}{c^2+a^2}}\geq \frac{1}{\sqrt{2}}$
Chiếu theo tư tưởng trên ta phải làm mất căn ở mẫu trên,nhưng bằng cách nào?Bình phương lên?đó không phải là lựa chọn tốt vì khối lượng tính toán sẽ không nhỏ?
Xét phân thức $\sqrt{\frac{ab}{a^2+b^2}}$
Ta sẽ nhân thêm$\sqrt{a^2+b^2}$ vào cả tử và mẫu
$\sqrt{\frac{ab}{a^2+b^2}}= \frac{\sqrt{ab(a^2+b^2)}}{a^2+b^2}$
Vẫn ở tư tưởng khử căn,chú ý đến BĐT quen thuộc,ta có $\sqrt{ab(a^2+b^2)}\geq \sqrt{2}ab$
Vậy,t phải chứng minh $\frac{\sqrt{2}ab}{a^2+b^2}+\frac{\sqrt{2}bc}{b^2+c^2}+\frac{\sqrt{2}ca}{c^2+a^2}\geq \frac{1}{\sqrt{2}}$
Hay $\frac{ab}{a^2+b^2}+\frac{bc}{b^2+c^2}+\frac{ca}{c^2+a^2}\geq \frac{1}{2}$( với $a^2+b^2+c^2=2(ab+bc+ca)$
(các bạn chứng minh hộ mình nhé :icon6: )
b,Nhân 1 lượng vừa đủ vào tử
Cũng chung ý tưởng như ở mục a,tuy nhiên,ở đây ta lại làm mất căn ở tủe số,và sau đó là đánh giá mẫu số bằng AM-GM
VD:Cho các số thực dương a,b,c có tổng bằng 3,chứng minh
$\sqrt{\frac{a^2+b^2+c}{a+b+c^2}}+\sqrt{\frac{b^2+c^2+a}{c+b+a^2}}+\sqrt{\frac{c^2+a^2+b}{c+a+b^2}}\geq 3$
Giải:Ta sẽ làm mất căn ở tử số bởi (bằng cách nhân lượng hợp lý-giống ở mục a)2 lí do sau
a,Sau khi nhân vào tử và mẫu,ta có thể đánh giá mẫu mà không làm đổi chiều bất đẳng thức
b,Xem dlời giải bạn sẽ hiểu
Theo AM-GM ta có $\sqrt{\frac{a^2+b^2+c}{a+b+c^2}}= \frac{a^2+b^2+c}{\sqrt{(a^2+b^2+c)(a+b+c^2)}}\geq \frac{2(a^2+b^2+c)}{a^2+b^2+c^2+a+b+c}$
Các bạn đẫ hiểu tại sao ta làm mất căn ở tử rồi đúng không?Sau khi dùng AM-GM,mẫu thức đã"xem như được đưa về giống nhau)
Xây dựng các BĐT tương tự,ta chỉ cần chứng minh
$\frac{4(a^2+b^2+c^2)+2)a+b+c)}{a^2+b^2+c^2+a+b+c}\geq 3$
hay $a^2+b^2+c^2\geq a+b+c$
với $a+b+c=3$ thì BĐT này không khó chứng mnih
c,Đặt ẩn phụ
Vì kĩ thuật đặt ăn phụ mình đã trình bày nên xin chỉ nêu 1 VD nho nhỏ sau
Chứng mnih với mọi a,b,c dương,ta có
$\frac{1}{a\sqrt{a+b}}+\frac{1}{b\sqrt{b+c}}+\frac{1}{c\sqrt{c+a}}\geq \frac{3}{\sqrt{2abc}}$
Giải BĐT cần chứng minh tương đương $\sqrt{\frac{2bc}{a(a+b)}}+\sqrt{\frac{2ca}{b(b+c)}}+\sqrt{\frac{2ab}{c(c+a)}}\geq 3$
Ta sẽ đặt $\sqrt{\frac{2bc}{a(a+b)}}=x;\sqrt{\frac{2ca}{b(b+c)}}=y;\sqrt{\frac{2ab}{c(c+a)}}=z$
Chú ý thêm rằng xy=$\frac{2c}{\sqrt{(a+b)(b+c)}}$,ta sẽ chứng minh $xy+yz+zx\geq 3$
Tiếp tục đặt $\sqrt{a+b}=p,\sqrt{b+c}=q,\sqrt{c+a}=r$,khi đó,dề thấy $xy=\frac{p^2+q^2-r^2}{qr}$
BĐT cần chứng minh sẽ là $(p^3+q^3+r^3)(p^2q+q^2r+r^2p)\geq q^2p+r^2q+p^2r+3pqr$(Đây là 1 BĐT khá hay,xin nhường cho các bạn)
d,Đánh giá trực tiếp bằng AM-GM
Mình cũng xin chỉ đưa ra 1 VD sau
Chứng minh với mọi số dương a,b,c có tổng bằng 3
$\sqrt{\frac{a+b}{c+ab}}+\sqrt{\frac{b+c}{a+bc}}+\sqrt{\frac{c+a}{b+ca}}\geq 3$
Giải: Áp dụng trực tiếp AM-GM 3 số,ta cần chứng minh
$(a+b)(b+c)(c+a)\geq (c+ab)(a+bc)(b+ca)$
Chú ý rằng theo AM-GM thì $(c+ab)(a+bc)\leq \left [ \frac{c+ab+a+bc}{2} \right ]^2=\frac{(b+1)^2(c+a)^2}{4}$
Xây dựng các BĐT tương tự,ta cần CM $(a+1)(b+1)(c+1)\leq 8$
tuy nhiên ,với a+b+c=3,BĐT trên hiển nhiên đúng
(Phần này sẽ được nới rõ ở kĩ thuật ghép đối xứng)
e,Khử mẫu gián tiếp bằng AM-GM để đưa về cùng mẫu
VD:chứng minh với mọi số thực dương a,b,c có $ab+bc+ca\geq 1$
$\frac{1}{\sqrt{a^2+ab+b^2}}+\frac{1}{\sqrt{b^2+bc+c^2}}+\frac{1}{\sqrt{c^2+ca+a^2}}\geq \frac{9}{(a+b+c)^2}$
Giải: Ta chú ý đến đánh giá sau
$(ab+bc+ca)(a^2+ab+b^2)\leq \frac{(a+b)^2(a+b+c)^2}{4}$
nhưng do $ab+bc+ca\geq 1\Rightarrow \sqrt{a^2+ab+b^2}\leq \frac{(a+b)(a+b+c)}{2}$ hay
$\frac{1}{\sqrt{a^2+ab+b^2}}\geq \frac{2}{(a+b)(a+b+c)}$
phần còn lại dc giải khá đơn giản,xin mời các bạn >:)
+,Làm mất mẫu
Tuy rằng các cách đã nêu trên khá "ngon lành" đối với các BĐT chứa căn thức ở mẫu nhưng xem ra lại khá trơ đối với các BĐT không căn thức ở mẫu.Trong trường hợp này,1 trong những cách có thể chọn là khử mẫu.Tư tưởng này đã dc nhắc đến ở phần mở đầu và có phần hơi động chạm tới cân bằng hệ số,tuy nhiên đây là 1 kĩ thuật buộc phải nắm vững
ta xét VD sau (IMO Shorlít 1998):Chứng minh với mọi x,y,z dương có tích bằng 1,ta có BĐT sau
A= $\frac{x^3}{(1+y)(1+z)}+\frac{y^3}{(1+z)(1+x)}+\frac{z^3}{(1+x)(1+y)}\geq \frac{3}{4}$
Giải: Ta sẽ làm gì với BĐT trên,nếu xử lý theo các phương pháp nêu trên thì xem chừng không có hiệu quả
Đánh gíá trung bình nhân-trung bình cộng là điều không thể thực hiện
Đặt ẩn phụ cũng sẽ rất khó khăn :wacko:
Ta phải tìm cách đánh giá làm sao để mất mẫu đi
Theo cách nghĩ như trên ,ta xét riêg với phân thức $\frac{x^3}{(1+y)(1+z)}$ (các phân thức khác sẽ đánh giá tương tự )
+,Trước tiên,ta kiểm tra (và thấy)BĐT trở thành Đẳng thức khi x=y=z
+,Việc của chúng ta là phải xem ta sẽ áp dụng AM-GM với bộ bao nhiêu số
Để ý điều kiện đề bài là tích 3 số (ở bậc 1),trong khi đó,số mũ của tử số là mũ 3,vì vậy,trong tư tưởng của ta,việc đánh giá phải làm mất được mẫu và đưa tử về bậc 1 (hoặc lớn hơn)-Tuy nhiên,chẳng dại gì mà ta lại đánh giá để đưa về bậc lớn,vì càng lớn thì việc khử "phần thêm" sẽ càng khó,Vậy,ta sẽ áp dụng cho 3 số(Sau này,trong hầu hết các bài toán đánh giá mẫu,thường thì bậc của tử là bậc bao nhiêu thì sẽ đánh giá cho từng ấy số hạng )
+,Đánh giá với những số hạng nào?(cái này mới quan trọng nè)
Ta đã có 1(trên 3)số hạng và phải tìm 2 số hạng còn lại .Tuy nhiên,việc tìm 2 số hạng này không khó(trên tư tưởng làm mất mẫu),2 số hạng cần tìm đương nhiên sẽ có dạng $\frac{1+y}{a},\frac{1+z}{b}$
+,Chỉ cần cho $x=y=z$ là ta tìm được a,b
+,Kết hợp với đề bài là ta có cái ta muốn
+,Sau đây là lời giải đầy đủ
Áp dụng BĐT AM-GM cho 3 số,ta có
$\frac{x^3}{(1+y)(1+z)}+\frac{1+y}{8}+\frac{1+z}{8}\geq \frac{3x}{4}$
Xây dựng các BĐT tương tự ,ta có $A\geq \frac{x+y+z}{2}-\frac{3}{4}$
Bây giờ,chỉ cần chú ý là với diều kiện xyz=1 thì $x+y+z\geq 3\sqrt[3]{xyz}=3$
BĐT xem như đã được giải quyết trọn vẹn
*Tuy nhiên,không dừng lại ở đây,ta còn có thẻ(theo những suy luận không phải không có căn cứ) cố gắng tìm 1 lời giải khác,sẽ có 2 hướng cơ bản sau
+,Điều kiện đề cho xyz=1, liệu ta có hi vọng biến $(x,y,z)\rightarrow (\frac{a}{b},\frac{b}{c},\frac{c}{a})$ không?(theo cách nghĩ của ẩn phụ)
+,Nhận thấy là mẫu chung của các phân thức là $(1+x)(1+z)(1+y)$ và việc đưa về mẫu chung như trên cũng không quá phức tạp,vậy liệu làm thế có cho ta lời giải khác không?
(Phần này xin dành cho những người thích tìm tòi -mình nhác quá :angry: )

Bài viết đã được chỉnh sửa nội dung bởi no matter what: 24-11-2012 - 19:42


#14
no matter what

no matter what

    Why not me

  • Thành viên
  • 397 Bài viết
Do phần này khá rộng nên mình xin trình bày bài tập riêng (hơi nhiều 1 tí) :icon6:
1,Chứng minh với mọi a,b,c $0\leq a,b,c\leq \frac{1}{2}$ thỏa mãn $a+b+c=1$ thì
$\frac{1}{a(2b+2c-1)}+\frac{1}{b(2c+2a-1)}+\frac{1}{c(2a+2b-1)}\geq 27$
2,Chứng minh với mọi a,b,c $\geq \frac{1}{2}$,ta có
$\frac{a^2}{\sqrt{5-2(b+c)}}+\frac{b^2}{\sqrt{5-2(c+a)}}+\frac{c^2}{\sqrt{5-2(a+b)}}\geq 3$
3,Chứng minh với mọi a,b,c có $0< a,b,c\leq \frac{1}{3},a^3+b^3+c^3=\frac{3}{64}$ thì
$\frac{1}{1-3a}+\frac{1}{1-3b}+\frac{1}{1-3c}\geq 12$
4,Chứng minh với mọi số dương a,b,c có tích bằng 1
$\frac{1}{2a^3+b^3+c^3+2}+\frac{1}{a^3+2b^3+c^3+2}+\frac{1}{a^3+b^3+2c^3+2}\leq \frac{1}{2}$
5,Với mọi x,y,z có $x+2y+3z=\frac{1}{4}$,tìm MAX
$\frac{232y^3-x^3}{2xy+24y^2}+\frac{783z^3-8y^3}{6yz+54z^2}+\frac{29x^3-27z^3}{3xz+6x^2}$
6,Chứng minh với mọi a,b,c không âm
$\sqrt{\frac{ab+bc+ca}{3}}\leq \sqrt[3]{\frac{(a+b)(b+c)(c+a)}{8}}$
7,Chứng minh với mọi số dương a,b,c có tích bằng 1
$\frac{bc}{a^2b+a^2c}+\frac{ca}{b^2a+b^2c}+\frac{ab}{c^2a+c^2b}\geq \frac{3}{2}$
8,Chứng minh với mọi số dương a,b,c có tổng bằng 3
$\frac{ab}{a+b+2c}+\frac{bc}{b+c+2a}+\frac{ca}{c+a+2b}\leq \frac{3}{4}$
9, Chứng minh với mọi a,b,c dương có $ab+bc+ca=3$
$\frac{1}{1+a^2(b+c)}+\frac{1}{1+b^2(c+a)}+\frac{1}{1+c^2(a+b)}\leq \frac{1}{abc}$
10,Chứng minh với mọi x,y,z $> -1$,ta có
$\frac{1+x^2}{1+y+z^2}+\frac{1+y^2}{1+z+x^2}+\frac{1+z^2}{1+x+y^2}\geq 2$

#15
no matter what

no matter what

    Why not me

  • Thành viên
  • 397 Bài viết
Bài viết 1
Trong lúc lục lọi trên google,mình tình cờ tìm ra bài viết này(cũng khá lâu rồi),mình thấy nó cũng khá hay nên up lên cho mọi người mong rằng ít nhiều giúp ích(bài này của bạn nào ở trường LÊ HỒNG PHONG ấy
(Ak,phải nói thêm là diễn đàn mình ai thick Sherlock holmes không nhỉ -bác tg bài này chắc cũng nghiện ổng-mình cũng là 1 fan)
Bất đẳng thức và sợi dây dài ràng buộc
"Đứng trên phương diện của một người hâm mộ Sherlock holmes,tôi tự cho phép mìnhđồng nhất các quan niệm về "quan sát": và"suy luận"của Holmes vào toán học:"Toàn bộ cuộc sống là một chuỗi mắc xích mà ta có thể hiểu được bản chất của nó nếu ta biết được một nắc xích",và hơn hết,sự liên kết đó góp phần quan trọng để tìm ra cách giải quyết của một vấn đề nào đó"Các vụ phạm pháp đều có nét giống nhau,nếu ta thuộc lòng các chi tiết của một vụ thì rất dễ ần ra đầu mối của vụ thứ 1001"
Có thẻ các bạn xem những lừoi trên là thừa thãi,vô ích,còn riêng tôi,nó lại có một sự tác động kì lạ
Xét bài toán sau
$\frac{a}{b+c}+\frac{b}{c+a}+\frac{c}{a+b}\geq \frac{3}{2}$
Nếu bạn đã quen với BĐT thì Nesbitt 3 biến đối với bạn chỉ là"trò trẻ",không sai,tôi không có lý do gì để phản bác
Cũng phải nói thêm là đã có khoảng 45 cách(con số không nhỏ"để chứng mnih BĐT đẹp đẽ kia.Tuy nhiên,mục đích của tôi không phải là đưa ra một đống cách giải.Cái tôi muốn nói ở đây là BĐT trên có rất nhiều ứng dụng
a,Hãy suy xét một bài toán cơ bản
Ta xét BĐT sau:Với mọi a,b,c dương có tổng bằng 3
$\frac{1}{a^2(1+a)}+\frac{1}{b^2(1+b)}+\frac{1}{c^2(1+c)}\geq \frac{3}{4abc}$(Olimpic 30-4)
Giải:Ta hoàn toàn có thể giải quyết bài toán trên bằng đánh giá thuận C-S,tuy nhiên,ta sẽ xem xét 1 lời giải sau
BĐT
$\Leftrightarrow \frac{abc}{a^2(1+a)}+\frac{abc}{b^2(1+b)}+\frac{abc}{c^2(1+c)}\geq \frac{3}{4}$
$\Leftrightarrow \frac{bc(b+c)}{a\left [ (b+c)^2+2a(b+c) \right ]} +\frac{ca(c+a)}{b\left [ (c+a)^2+2b(c+a) \right ]}+\frac{ab(a+b)}{c\left [ (a+b)^2+2c(a+b) \right ]}\geq \frac{3}{4}$
$\Leftrightarrow \frac{bc(b+c)}{2a\left [ (b^2+c^2+a(b+c) \right ]} +\frac{ca(c+a)}{2b\left [ a^2+c^2+b(c+a) \right ]}+\frac{ab(a+b)}{2c\left [ a^2+b^2+c(a+b) \right ]}\geq \frac{3}{4}$
$\Leftrightarrow \frac{(b+c)}{2a(\frac{a+b}{c}+\frac{a+c}{b})} +\frac{(c+a)}{2b(\frac{b+c}{a}+\frac{a+b}{c})}+\frac{(a+b)}{2c(\frac{b+c}{a}+\frac{c+a}{b})}\geq \frac{3}{4}$

Bằng việc đặt $\frac{a+b}{c}=x,\frac{b+c}{a}=y,\frac{c+a}{b}=z$,ta dễ dàng đưa BĐT trên về nesbitt 3 biến
VD2: Chứng minh với mọi a,b,c thực dương thì
$\frac{a}{(b+c)^2}+\frac{b}{(c+a)^2}+\frac{c}{(a+b)^2}\geq \frac{9}{4(a+b+c)}$
Giải :BĐT cần chứng minh tương đương
$(a+b+c)(\frac{a}{(b+c)^2}+\frac{b}{(c+a)^2}+\frac{c}{(a+b)^2})\geq \frac{9}{4}$
tuy nhiên,theo C-S,ta có $(a+b+c)(\frac{a}{(b+c)^2}+\frac{b}{(c+a)^2}+\frac{c}{(a+b)^2})\geq (\frac{a}{b+c}+\frac{b}{c+a}+\frac{c}{a+b})^2$
Phần việc còn lại là của Nesbitt
b,Chùm bài Tương tự nhau
Xét bài toán sau:Cho a,b,c thực dương có tích bằng 1,chứng minh
$\frac{1}{a^3+b+c}+\frac{1}{b^3+c+a}+\frac{1}{c^3+a+b}\leq \frac{3}{a+b+c}$
Giải:Dựa ccào mẫu thức,ta định hướng đưa bài toán về cùng mẫu,nhưng để làm điều này,ta không dùng AM-GM,đây là nhiệm vụ của Holder
Áp dụng BĐT Holder,ta có $(a^3+b+c)(1+b+c)(1+b+c)\geq (a+b+c)^3$
Vậy,với abc=1,ta chỉ cần chứng minh cho $(1+b+a)^2+(1+b+c)^2+(1+c+a)^2\leq 3(a+b+c)^2$
hay $a^2+b^2+c^2+4(ab+bc+ca)\geq 3+4(a+b+c)$
Thực hiện đổi biến p,q,r ,BĐT tương dương$p^2-2q-4p+3\geq 0$
Với r=1 thì $p\geq 3;q\geq 3$

BĐT tương đương $(p-3)^2+2(p+q-6)\geq 0$(ĐÚNG)

Ta sẽ xem xét thêm 1 BĐT có chứa căn thúc sau
Với mọi số thực dương a,b,c có tổng bình phương bằng 3,ta có BĐT
$\sqrt{\frac{a^2}{a^2+b+c}}+\sqrt{\frac{b^2}{b^2+c+a}}+\sqrt{\frac{c^2}{c^2+a+b}}\leq \sqrt{3}$
(Ukraine 2008)
Giải:Áp dụng C-S cho mẫu $(a^2+b+c)(1+b+c)\geq (a+b+c)^2$
Suy ra $\sqrt{\frac{a^2}{a^2+b+c}}\leq \frac{a\sqrt{1+b+c}}{a+b+c}$
Xây dsựng các BĐT tương tự,ta có
VT$\leq \frac{a\sqrt{1+b+c}+b\sqrt{1+c+a}+c\sqrt{1+a+b}}{a+b+c}$
$\frac{a\sqrt{1+b+c}+b\sqrt{1+c+a}+c\sqrt{1+a+b}}{a+b+c}\leq$
$\frac{\sqrt{(a+b+c)\left [ a(1+b+c)+b(1+c+a)+c(1+a+b) \right ]}}{a+b+c}$
=$\frac{\sqrt{(a+b+c)^2+(a+b+c)2(ab+bc+ca)}}{a+b+c}$
=$\sqrt{1+\frac{2(ab+bc+ca)}{a+b+c}}\leq \sqrt{1+\frac{2(a+b+c)}{3}}\leq \sqrt{1+\frac{2\sqrt{3(a^2+b^2+c^2)}}{3}}= \sqrt{3}$

Hơn nữa,ta còn cs bài toán khó hơn
Chứng minh với a,b,c thực dương có $a+b+c+1=4abc$,ta có
$\frac{1}{a^4+b+c}+\frac{1}{b^4+c+a}+\frac{1}{c^4+a+b}\leq \frac{3}{a+b+c}$
Bài này xin để dành cho các bạn
c,Làm mạnh BĐT
Việc làm mạnh BĐT xuất hiện khá tình cờ trong khi thực hiện các phép phân tách.Tuy nhiên,các BĐT sau khi làm mạnh thường rất khó và lời giải cho nó cũng thường phải nhờ vào các cộng cụ mạnh như.SO.S,SMV,...
Sau đây là 1 số ví dụ
1,Một hệ quả quên thuộc của Schwaz:
$\frac{1}{a}+\frac{1}{b}+\frac{1}{c}\geq \frac{9}{a+b+c}$
Và bạn hãy tìm lời giải của BĐT sau $\frac{1}{a}+\frac{1}{b}+\frac{1}{c}\geq \frac{4(a+b+c)^2}{ab+bc+ca}-3$
BĐT trên chưa cần tới các công cụ mạnh,ta có thể giải khá vô tư bằng C.S
BĐT tương đương $(\frac{a}{b}+\frac{b}{a}+2)+(\frac{a}{c}+\frac{c}{a}+2)(\frac{c}{b}+\frac{b}{c}+2)\geq \frac{4(a+b+c)^2}{ab+bc+ca}$
việc còn lại chỉ là thực hiện phép tính và C-S -E
VD2: Bắt đầu từ BĐT $\frac{a^2}{b}+\frac{b^2}{c}+\frac{c^2}{a}\geq a+b+c$,ta có thể đưa ra các BĐT mạnh hơn sau
a1:$\frac{a^2}{b}+\frac{b^2}{c}+\frac{c^2}{a}\geq a+b+c+\frac{4(a-b)^2}{a+b+c}$
a2:$\frac{a^2}{b}+\frac{b^2}{c}+\frac{c^2}{a}\geq a+b+c+\frac{1}{b+c}(a-b)^2+\frac{1}{c+a}(b-c)^2+\frac{1}{a+b}(c-a)^2$
Tư tưởng giải 2 bài này giống như bài trên nên các bạn luyện tập thêm
"Cũng chẳng có gì khó cả,cái này bình thường thôi"-tôi tin là bạn đang nghĩ vậy,tuy nhiên,bạn hãy xem thêm 2 VD sau để chắc chắn cái bạn nói là đúng nhé
Từ BĐT AM-GM 3 số,ta có thể làm mạnh lên thành
$\frac{a+b+c}{3}\geq \sqrt[3]{abc}+\frac{(a-b)^2+(b-c)^2+(c-a)^2}{12(a+b+c)}$
Cũng cần nói thêm là BĐT này cần tới S.O.S để giải (quá ảo) :lol:

Bài viết đã được chỉnh sửa nội dung bởi no matter what: 24-11-2012 - 13:13


#16
ckuoj1

ckuoj1

    Trung sĩ

  • Thành viên
  • 177 Bài viết
Câu 10(BĐT Schur:Dưới đây là một số cách chứng minh BĐT)
http://tailieu.vn/xe...hur.296127.html
Những người thông minh là những người biết bị thần kinh đúng lúc ^^

#17
NGUYEN MINH HIEU TKVN

NGUYEN MINH HIEU TKVN

    Binh nhất

  • Thành viên
  • 29 Bài viết
Anh ơi, anh chịu khó gửi thành tập tài liệu fdf cho chúng em nhé!!!
Anh( chị ) nào đọc xong cái này del hộ em cái!!!

#18
minhtuyb

minhtuyb

    Giả ngu chuyên nghiệp

  • Thành viên
  • 470 Bài viết

*Mình xin dừng phần này ở đây đã
Một số bài tập nho nhỏ cho các bạn
1, chứng minh với mọi a,b,c không âm có $ab+bc+ca+abc=4$ thì
$a+b+c\geq ab+bc+ca$(VMO 1996)
2,Chứng minh với mọi a,b,c dương có $ab+bc+ca+2abc=1$ thì
$2(a+b+c)+1\geq 32abc$

Bài $VMO\ 1996$ thì đã quá quen thuộc rồi mình xin không c/m lại. Còn bài $2$ thì ta cần đánh giá kĩ thuật một chút (xuất phát từ đẳng thức):
---
-Xét $abc=0$ thì BĐT đã cho hiển nhiên đúng
-Xét $abc\ne 0:$
BĐT đã cho tương đương:
$$\dfrac{2(a+b+c)+1}{2abc}\ge 16\\ \Leftrightarrow \dfrac{1}{ab}+\dfrac{1}{bc}+\dfrac{1}{ca}+\dfrac{1}{2abc}\ge 16$$
Đến đây bài toán đã trở nên khá đơn giản, áp dụng BĐT $\sum\limits_{x,y,z,t} \dfrac{1}{x}\ge \dfrac{16}{x+y+z+t}$ với $x,y,z$ dương ta có đpcm
---
Mình nghĩ bài toán này được chế từ đẳng thức $\dfrac{1}{ab}+\dfrac{1}{bc}+\dfrac{1}{ca}+\dfrac{1}{2abc}= \dfrac{2(a+b+c)+1}{2abc}$. Vậy tại sao các bạn ko thử tự chế cho mình vài BĐT khác ? :wub:

P/s: Không nhất thiết là bài nào cũng đổi biến rồi làm trâu nhỉ? :icon6:

Bài viết đã được chỉnh sửa nội dung bởi minhtuyb: 24-11-2012 - 22:43

Phấn đấu vì tương lai con em chúng ta!

#19
no matter what

no matter what

    Why not me

  • Thành viên
  • 397 Bài viết

Bài $VMO\ 1996$ thì đã quá quen thuộc rồi mình xin không c/m lại. Còn bài $2$ thì ta cần đánh giá kĩ thuật một chút (xuất phát từ đẳng thức):
---
-Xét $abc=0$ thì BĐT đã cho hiển nhiên đúng
-Xét $abc\ne 0:$
BĐT đã cho tương đương:
$$\dfrac{2(a+b+c)+1}{2abc}\ge 16\\ \Leftrightarrow \dfrac{1}{ab}+\dfrac{1}{bc}+\dfrac{1}{ca}+\dfrac{1}{2abc}\ge 16$$
Đến đây bài toán đã trở nên khá đơn giản, áp dụng BĐT $\sum\limits_{x,y,z,t} \dfrac{1}{x}\ge \dfrac{16}{x+y+z+t}$ với $x,y,z$ dương ta có đpcm
Mình nghĩ bài toán này được chế từ đẳng thức $\dfrac{1}{ab}+\dfrac{1}{bc}+\dfrac{1}{ca}+\dfrac{1}{2abc}= \dfrac{2(a+b+c)+1}{2abc}$. Vậy tại sao các bạn ko thử tự chế cho mình vài BĐT khác ? :wub:

nhìn lời giải này mà mình thấy rầy quá :ohmy: ,bài này sau khi đổi ẩn mình còn nhân tung tóe ra mới đau chứ? :icon10:

#20
no matter what

no matter what

    Why not me

  • Thành viên
  • 397 Bài viết
ĐÈ NGHỊ 2
Chứng minh với mọi a,b,c $0\leq a,b,c\leq \frac{1}{2}$ thỏa mãn $a+b+c=1$ thì
$\frac{1}{a(2b+2c-1)}+\frac{1}{b(2c+2a-1)}+\frac{1}{c(2a+2b-1)}\geq 27$
(Bài đã nêu ở mục 3(Rất tiếc đề nghị 1 vẵn chưa ai giải) :angry:




0 người đang xem chủ đề

0 thành viên, 0 khách, 0 thành viên ẩn danh